-
Notifications
You must be signed in to change notification settings - Fork 0
/
Copy pathtemple.tex
42 lines (38 loc) · 2.04 KB
/
temple.tex
1
2
3
4
5
6
7
8
9
10
11
12
13
14
15
16
17
18
19
20
21
22
23
24
25
26
27
28
29
30
31
32
33
34
35
36
37
38
39
40
%arara: xelatex
\documentclass{ctexart}
\usepackage{amsmath,amssymb,amsthm,bbm}
\newtheorem{definition}{Definition}
\newtheorem{theorem}{Theorem}
\everymath{\displaystyle}
\newlength\inlineHeight
\newlength\inlineWidth
\long\def\(#1\){%
\settoheight{\inlineHeight}{$#1$}%
\settowidth{\inlineWidth}{$#1$}%
\ifdim \inlineWidth > 0.5\textwidth%
$$#1$$%
\else%
\ifdim \inlineHeight > 1.5em%
$$#1$$%
\else%
$#1$%
\fi%
\fi%
}
\begin{document}
\begin{solution}
Since \(M\) is orientable, we can assume that \(\mathcal{B} \subset \mathcal{A}\) is an oriention of \(M\), where \(\mathcal{A}\) is all local coordinate of \(M\).
Now consider \(\mathcal{C}:=\{(U;-x^i):(U;x^i) \in \mathcal{B}\}\). Easily to check that \(\mathcal{C}\) is an oriention of \(M\), too.
And obviously \(\mathcal{B} \cap \mathcal{C} = \varnothing\), thus \(\mathcal{B} \neq \mathcal{C}\). So there is two oriention. Now we need to prove there is no other oriention.
Assume \(\mathcal{D}\) is an oriention of \(M\). We will define a function \(\sgn:M \to \{1,-1\}\) by
\(\sgn(p)=1 \iff \exists (U_0,x_0^i) \in \mathcal{B},\exists (V_0,y_0^i) \in \mathcal{D},p \in U_0 \cap V_0,J_{x_0,y_0}(p)>0\).
We will prove that \(\sgn(p)=1 \implies \forall (U,x^i) \in \mathcal{B},\forall (V;y^i) \in \mathcal{D},p \in U \cap V \implies J_{x,y}(p)>0\).
It's easy because \(J_{x,y}=J_{x,x_0}J_{x_0,y_0}J_{y_0,y}\), and \(J_{x,x_0}>0,J_{y,y_0}>0\) by definition of oriention.
So \(\sgn(p)=-1 \iff \exists (U_0,x_0^i) \in \mathcal{C},\exists (V_0,y_0^i) \in \mathcal{D},p \in U_0 \cap V_0,J_{x_0,y_0}(p)>0 \)
\(\iff \forall (U,x^i) \in \mathcal{B},\forall (V;y^i) \in \mathcal{D},p \in U \cap V \implies J_{x,y}(p)>0 \).
Noting that if \(\sgn(p)=1\), we have \(\forall q \in U_0 \cap V_0,\sgn(q)=1\), and so is \(\sgn(p)=-1\).
So \(\sgn\) is continuous. So \(\sgn\) is constant because \(M\) is connected.
Easy to check that \(\sgn(p)=1 \iff \mathcal{B}=\mathcal{D}\), and \(\sgn(p)=-1 \iff \mathcal{C}=\mathcal{D}\).
So there is only two oriention of \(M\).
\end{solution}
\end{document}